Diễn Đàn MathScopeDiễn Đàn MathScope
  Diễn Đàn MathScope
Ghi Danh Hỏi/Ðáp Community Lịch

Go Back   Diễn Đàn MathScope > Sơ Cấp > Tài Liệu > Đề Thi > Đề Thi HSG Cấp Tỉnh ở Việt Nam

News & Announcements

Ngoài một số quy định đã được nêu trong phần Quy định của Ghi Danh , mọi người tranh thủ bỏ ra 5 phút để đọc thêm một số Quy định sau để khỏi bị treo nick ở MathScope nhé !

* Nội quy MathScope.Org

* Một số quy định chung !

* Quy định về việc viết bài trong diễn đàn MathScope

* Nếu bạn muốn gia nhập đội ngũ BQT thì vui lòng tham gia tại đây

* Những câu hỏi thường gặp

* Về việc viết bài trong Box Đại học và Sau đại học


Trả lời Gởi Ðề Tài Mới
 
Ðiều Chỉnh Xếp Bài
Old 01-10-2013, 06:29 AM   #1
namdung
Administrator

 
Tham gia ngày: Feb 2009
Đến từ: Tp Hồ Chí Minh
Bài gởi: 1,343
Thanks: 209
Thanked 4,066 Times in 778 Posts
Gửi tin nhắn qua Yahoo chát tới namdung
Đề thi các trường và các tỉnh năm học 2013-2014 - Lời giải và bình luận

Hôm nay là 1/10 và mùa chọn đội tuyển cho VMO 2014 ở các trường và các tỉnh đã thực sự khởi động. Đà Nẵng đã lập xong đội tuyển, Cần Thơ, Đồng Tháp, PTNK, KHTN HN ... đã thi vòng 1 và cả vòng 2 ...

Tôi tiếp tục mở chuyên mục này để chúng ta thảo luận về các đề thi ở các trường, các tỉnh năm nay, nhằm đem đến cho các bạn học sinh đang chuẩn bị cho VMO 2014 không chỉ là đáp án, lời giải, mà còn là những bình luận về phương pháp áp dụng trong các bài toán đó, xuất xứ của bài toán và những bài toán tương tự.

Như thường lệ, chúng ta sẽ chia các bài toán thành các chủ đề sau:

1. Bất đẳng thức
2. Dãy số và giới hạn
3. Hàm số, đa thức & phương trình hàm
4. Phương trình và hệ phương trình
5. Số học
6. Tổ hợp
7. Hình học
8. Một số bài toán khác

Tôi và các cộng sự sẽ tập hợp các đề toán theo từng chủ đề, gửi lên đây và chúng ta có thể vào giải và bình luận. Có thể bình luận trực tiếp trong chủ đề này hoặc là gửi file đính kèm.

Năm 2014 sẽ là một năm tròn của nhiều sự kiện quan trọng: 50 năm ra số báo THTT đầu tiên, 40 năm Việt Nam tham dự kỳ thi toán quốc tế ... Vì thế, hứa hẹn năm 2014 sẽ là một năm của nhiều thành công cho toán học Việt Nam nói chung và phong trào chuyên toán nói riêng.

Chúc các bạn học sinh một mùa ôn luyện nhiều niềm vui và thành công. Chúc các thầy cô giáo có nhiều sức khỏe, niềm vui và nhiệt huyết để tiếp tục dẫn dắt các đàn em gặt hái những thành công.
[RIGHT][I][B]Nguồn: MathScope.ORG[/B][/I][/RIGHT]
 
namdung is offline   Trả Lời Với Trích Dẫn
The Following 26 Users Say Thank You to namdung For This Useful Post:
Akira Vinh HD (24-11-2014), babysama (22-08-2014), CHUNG-ĐTH (14-10-2013), cool hunter (06-10-2013), DaiToan (01-10-2013), doanthanh (01-10-2013), dungtoank22 (01-10-2013), henry0905 (02-10-2013), hoangqnvip (02-10-2013), huyt2k22 (30-10-2013), luugiangnam (01-10-2013), luxubuhl (01-10-2013), nguyentatthu (01-10-2013), NguyenThanhThi (01-10-2013), Nick Trần (01-10-2013), pco (01-10-2013), quocbaoct10 (01-10-2013), quoctri (02-10-2013), Raul Chavez (03-10-2013), Saruka 01 (27-10-2013), thaygiaocht (01-10-2013), thiendienduong (01-10-2013), Thmcuongvn (08-02-2014), transonlvt (11-11-2013), Trànvănđức (01-10-2013), trungno (06-11-2013)
Old 01-10-2013, 08:31 AM   #2
namdung
Administrator

 
Tham gia ngày: Feb 2009
Đến từ: Tp Hồ Chí Minh
Bài gởi: 1,343
Thanks: 209
Thanked 4,066 Times in 778 Posts
Gửi tin nhắn qua Yahoo chát tới namdung
Tập hợp 10 bài Tổ hợp.

Các bạn vào giải và bình luận nhé.


[RIGHT][I][B]Nguồn: MathScope.ORG[/B][/I][/RIGHT]
 
File Kèm Theo
Kiểu File : doc 2VMO2014_Tohop1.doc (51.5 KB, 738 lần tải)
namdung is offline   Trả Lời Với Trích Dẫn
The Following 13 Users Say Thank You to namdung For This Useful Post:
caubemetoan96 (30-10-2013), doanthanh (01-10-2013), mathandyou (01-10-2013), pco (01-10-2013), quocbaoct10 (01-10-2013), Raul Chavez (03-10-2013), sang_zz (17-10-2013), Saruka 01 (27-10-2013), thaygiaocht (01-10-2013), thiendienduong (01-10-2013), Trànvănđức (01-10-2013), trungno (06-11-2013), vuong_pn (02-10-2013)
Old 01-10-2013, 01:02 PM   #3
pco
+Thành Viên+
 
pco's Avatar
 
Tham gia ngày: Dec 2011
Bài gởi: 528
Thanks: 560
Thanked 195 Times in 124 Posts
Bài 6. Chứng minh rằng trong 39 số tự nhiên liên tiếp bất kỳ luôn có ít nhất một số có tổng các chữ số chia hết cho 11.
Lời giải. Trong $39$ số đã cho luôn tồn tại một số có chữ số tận cùng là $0$. Giả sử số đó là $\overline{a_1a_2 \cdots a_n0} \; (0 \le a_i \le 9, \; i= \overline{1,n}, a_1 \ne 0)$ và $a_1+a_2 + \cdots + a_n \equiv k \pmod{11}$.
Nếu $a_n=0$ thì xét các số $\overline{a_1a_2 \cdots a_{n-1}00}, \overline{a_1a_2 \cdots a_{n-1}01}, \cdots , \overline{a_1a_2 \cdots a_{n-1}10}$ chia cho $11$ dư $k,k+1,k+2, \cdots k+10$. Trong $11$ số này ắt hẳn sẽ có một số chia hết $11$. Vậy kết luân bài toán được chứng minh.

Nếu $a_n=9$ thì ta xét các số $\overline{a_1a_2 \cdots a_{n-1}89}, \overline{a_1a_2 \cdots a_{n-1}90}, \overline{a_1a_2 \cdots a_{n-1}91}, \cdots , \overline{a_1a_2 \cdots a_{n-1}99}$ tương tự ta cũng thu được kết luận bài toán.

Nếu $1 \le a_n \le 8$ thì ta xét các số $\overline{a_1a_2 \cdots (a_n-1)9}, \overline{a_1a_2 \cdots a_n0}, \overline{a_1a_2 \cdots a_n1}, \cdots , \overline{a_1a_2 \cdots a_9}$. Tương tự như trên ta cũng thu được kết luận bài toán

Vậy bài toán được chứng minh.

Nhận xét. Thực chất việc người ra đề đưa giả thiết "tổng các chữ số" chỉ là để đánh lừa con mắt người giải bài. Ta chỉ cần vận dụng tính chất cơ bản sau: Trong $n$ số tự nhiên liên tiếp luôn tồn tại một số chia hết cho $n$.
------------------------------
2. Cho dãy các phân số: 1/1, 1/2, 1/3, 1/4,..., 1/2012, 1/2013. Người ta biến đổi dãy bằng cách xóa đi 2 số a, b bất kỳ và thay bằng số a+b+ab. Sau 1 lần biến đổi số các số hạng giảm đi 1 đơn vị so với dãy trước. Chứng minh rằng giá trị của số hạng cuối sau 2012 lần biến đổi không phụ thuộc vào thứ tự thực hiện và tìm giá trị đó.

Lời giải. Ta đi làm phép thử, thay số $a,b$ trong dãy bởi số $ab+a+b=(a+1)(b+1)-1$. Gỉa sử ta lấy hai số $(a+1)(b+1)-1$ và số $c$ trong dãy ra và thay bằng $(a+1)(b+1)-1+c+c[(a+1)(b+1)-1]=(a+1)(b+1)(c+1)-1$.
Nếu ta tiếp tục thay số $(a+1)(b+1)(c+1)-1$ và số $d$ thì số đưa vào là $(a+1)(b+1)(c+1)(d+1)-1$.
Như vậy, sau $2012$ lần biến đổi sẽ còn lại số $\left( \frac{1}{1}+1 \right) \left( \frac{1}{2}+1 \right) \cdots \left( \frac{1}{2013}+1 \right)-1= \boxed{2013}$.

Nhận xét. Cái mấu chốt là $ab+a+b=(a+1)(b+1)-1$.
[RIGHT][I][B]Nguồn: MathScope.ORG[/B][/I][/RIGHT]
 
__________________
"People's dreams... will never end!" - Marshall D. Teach.

thay đổi nội dung bởi: pco, 01-10-2013 lúc 06:28 PM Lý do: Tự động gộp bài
pco is offline   Trả Lời Với Trích Dẫn
The Following 6 Users Say Thank You to pco For This Useful Post:
Akira Vinh HD (24-11-2014), giabao185 (09-10-2013), namdung (01-10-2013), quocbaoct10 (01-10-2013), Raul Chavez (03-10-2013), trungno (06-11-2013)
Old 01-10-2013, 02:28 PM   #4
nguyentatthu
Super Moderator
 
Tham gia ngày: Nov 2007
Đến từ: BH
Bài gởi: 212
Thanks: 135
Thanked 345 Times in 92 Posts
Tiếp theo là phần dãy số.
[RIGHT][I][B]Nguồn: MathScope.ORG[/B][/I][/RIGHT]
 
File Kèm Theo
Kiểu File : doc Dayso_VMO2014.doc (112.0 KB, 758 lần tải)
nguyentatthu is offline   Trả Lời Với Trích Dẫn
The Following 5 Users Say Thank You to nguyentatthu For This Useful Post:
luxubuhl (01-10-2013), namdung (01-10-2013), pco (01-10-2013), Raul Chavez (03-10-2013), vuong_pn (02-10-2013)
Old 01-10-2013, 04:21 PM   #5
Ispectorgadget
+Thành Viên+
 
Tham gia ngày: Dec 2011
Đến từ: Hồ Chí Minh city
Bài gởi: 98
Thanks: 53
Thanked 126 Times in 57 Posts
3. Yên Bái
Cho dãy $(x_n)$ xác định như sau\[\left\{ \begin{array}{l}
{x_1} = 1\\
{x_{n + 1}} = \left[ {\frac{5}{2}{x_n}} \right]
\end{array} \right.\,\,\forall n \ge 1\]
Chứng minh rằng dãy $(x_n)$ có vô hạn các số chẵn, có vô hạn các số lẻ. (Ký hiệu $[x]$ là phần nguyên của $x$)
Giải

Giả sử dãy $(x_n)^{\alpha}$ chỉ có hữu hạn các số chẵn, suy ra có ít nhất một số $n\in \mathbb{N}$ sao cho $x_k$ lẻ $\forall k\ge n$

Đặt: $x_k=2^{\alpha}.\beta +1$
$\alpha ,\beta \in \mathbb{N};\beta \text{lẻ}$

$x_{k+1} =2^{\alpha -1}5\beta+1 $

$x_{k+1}=2^{\alpha-2}5^2\beta+1 $

$....... $
$x_{k+\alpha} =5^{\alpha}\beta+1 $

$\Rightarrow x_{k+\alpha} \text{là số chẵn}\Rightarrow \text{vô lý}$

Từ đó suy ra rằng dãy đã cho phải có vô hạn các số chẵn.

Giả sử dãy $(x_n)^{\alpha}$ chỉ có hữu hạn các số lẻ,suy ra có ít nhất một số $n\in \mathbb{N}$ sao cho $x_n$ chẵn, $\forall k\ge n$

Đặt $x_k=2^{\alpha}.\beta +1$

Với $\alpha ,\beta \in \mathbb{N}
\beta $ lẻ .

$x_{k+1} =2^{\alpha -1}5\beta+1 $
$x_{k+1}=2^{\alpha-2}5^2\beta+1 $
$....... $
$x_{k+\alpha} =5^{\alpha}\beta+1 $
$\Rightarrow x_{k+\alpha} \text{là số lẻ}\Rightarrow \text{vô lý}$

Từ đó suy ra dãy có vô hạn số lẻ. $\square$
[RIGHT][I][B]Nguồn: MathScope.ORG[/B][/I][/RIGHT]
 
__________________
$F\begin{Bmatrix}
\heartsuit
\end{Bmatrix}=\frac{1}{\sqrt{2\pi}}\int_{-\infty }^{+\infty }f(t)e^{it\heartsuit}dt=? $
Ispectorgadget is offline   Trả Lời Với Trích Dẫn
The Following 4 Users Say Thank You to Ispectorgadget For This Useful Post:
1110004 (03-10-2013), Akira Vinh HD (24-11-2014), Raul Chavez (03-10-2013), trungno (06-11-2013)
Old 01-10-2013, 05:47 PM   #6
luxubuhl
+Thành Viên+
 
Tham gia ngày: Nov 2011
Bài gởi: 253
Thanks: 115
Thanked 121 Times in 63 Posts
Dãy số - Đồng Tháp

Dạng khá quen thuộc, ta xử lý từng phần như sau.

a) Ta có $a_{n+1}-a_n=\frac{a_n^2}{2013}>0$, suy ra $a_{n+1}>a_n$. Từ đó suy ra dãy đã cho là dãy tăng.

Dễ có $a_n>a_1=\frac{1}{2}$. Giả sử dãy đã có giới hạn là $L$, cho $n \to +\infty$ ta có $L=0$. Vô lý, vậy dãy không có giới hạn hữu hạn.

Vậy $a_n \to +\infty$.

b) Biến đổi dãy đã cho như sau

$$a_{n+1}=\frac{a_n^2+2013a_n}{2013}=\frac{a_n(a_n +2013)}{2013}$$

Suy ra $\frac{1}{a_n+2013}=\frac{1}{a_n}-\frac{1}{a_{n+1}}$, lấy tổng ta được

$$\lim \sum \frac{1}{a_n+2013}=\frac{1}{a_1}-\frac{1}{a_{n+1}}=2$$

Vì dãy $a_n \to +\infty$.
------------------------------
Dãy số - Chuyên Thăng Long ( Đà Lạt )

Dễ thấy $u_n>0$, $u_1=2012>\sqrt{2013}$ áp dụng BĐT $AM-GM$ có

$u_{n+1} \geq \frac{2\sqrt{2013}u_n}{2u_n}=\sqrt{2013}$

Suy ra $u_n \geq \sqrt{2013}$

Xét $f(x)=\frac{x^2+2013}{2x}$, có $f'(x)=\frac{2(x^2-2013)}{4x^2}$, suy ra $f$ đồng biến trên $(\sqrt{2013};+\infty)$.

Dễ thấy $u_2<u_1$, bằng quy nạp, dễ dàng chỉ ra rằng $u_n$ giảm, và bị chặn dưới, suy ra $u_n$ có giới hạn hữu hạn.

Đặt giới hạn hữu hạn của $u_n$ là $L$, cho $n \to +\infty$, suy ra $L=\sqrt{2013}$.
------------------------------
Dãy số - Bắc Giang - Copyright by huynhcongbang

Trích:
Nguyên văn bởi huynhcongbang View Post
Bài dãy số cũng là một dạng cũ $u_{n+1}=f(u_n, u_{n-1})$ nhưng ở đây thú vị hơn một tí là hàm $f$ có chứa phân thức chứ không phải dạng hàm hữu tỉ thông thường. Tuy nhiên, cũng nhờ phân thức như thế nên có nhiều lập luận sẽ dễ dàng hơn mà không cần xét dãy phụ như bài toán dạng gốc.

Ta thấy rằng $0 \le u_n < 2$ với mọi $n$.

Ta cũng thấy rằng hàm số $g(x)=\frac{3x+2}{2x+2+a}$ đồng biến với mọi $a \ge 0,x \in [0;2]$. Đây là điểm mấu chốt của bài toán này.

Ta có:
$x_3 = \frac{1}{6} > x_1$, $x_4 = \frac{15}{17} < x_2$ nên

$x_5 = \frac{3x_3+2}{2x_3+2+10x_4} > \frac{3x_1+2}{2x_1+2+10x_4} > \frac{3x_1+2}{2x_1+2+10x_2} = x_3$ và $x_6 = \frac{3x_4+2}{2x_4+2+10x_5} < \frac{3x_2+2}{2x_2+2+10x_5} < \frac{3x_2+2}{2x_2+2+10x_3} = x_4$

Do đó $x_1 < x_3 < x_5 <...$ và $x_2 > x_4 > x_6 >...$, nghĩa là bằng quy nạp, ta chứng minh được dãy $(x_{2n+1})$ tăng còn dãy $(x_{2n})$ giảm.

Cả hai dãy này đều bị chặn nên có giới hạn, gọi giới hạn của chúng lần lượt là $a, b \in [0;2]$.

Ta có:
$x_{2n+1} = \frac{3x_{2n-1}+2}{2x_{2n-1}+2+10x_{2n}} $ và $x_{2n+2} = \frac{3x_{2n}+2}{2x_{2n}+2+10x_{2n+1}} $
nên khi chuyển qua giới hạn thì ta được HPT:
$a = \frac{3a+2}{2a+2+10b}$ và $b = \frac{3b+2}{2b+2+10a}$.

Dễ dàng chứng minh được HPT này có nghiệm là $a=b=\frac{1+\sqrt{97}}{24} \in [0;2]$.

Vậy dãy số đã cho có giới hạn và giới hạn đó là $\lim x_n = \frac{1+\sqrt{97}}{24}$.
------------------------------
Dãy số - Chuyên Quảng Bình

Cho a là số thực dương tùy ý. Xét dãy số $({{x}_{n}})$ được xác định như sau:
${{x}_{1}}=a\,;{{x}_{n+1}}=\frac{{{x}_{n}}\sqrt{2+ \sqrt{2+...+\sqrt{2}}}}{{{x}_{n}}+1}$, (tử số có n dấu căn); $\forall n=1,2,3...$
Tính giới hạn của dãy số $({{x}_{n}})$.

Lời giải:

Trước hết, ta quy nạp được rằng $$u_n=\sqrt{2+\sqrt{2+\sqrt{2+....+\sqrt{2}}}}=2 \cos \frac{\pi}{2^{n+1}}$$

Quay lại bài toán, dãy có dạng

$$x_{n+1}=\frac{2x_n.\cos \frac{\pi}{2^{n+1}}}{x_n+1}$$

Đặt $a_n=\frac{1}{x_n}$, ta có $$a_{n+1}=\frac{a_n}{2\cos \frac{\pi}{2^{n+1}}}+\frac{1}{2\cos \frac{\pi}{2^{n+1}}}$$

Đặt $b_n=\frac{4a_n}{\sin \frac{\pi}{2^n}}$, được $$b_{n+1}=b_n+\frac{4}{\sin \frac{\pi}{2^n}}$$

Để ý rằng $\cot \frac{\pi}{2^{n+1}}-\cot \frac{\pi}{2^n}=\frac{1}{\sin \frac{\pi}{2^n}}$, từ đó suy ra $$b_n-4\cot \frac{\pi}{2^n}=....=b_1-4\cot\frac{\pi}{2}=b_1$$

Do đó $a_n=\frac{b_1 \sin \frac{\pi}{2^n}}{4}+\cos \frac{\pi}{2^n}$, cho $n \to \infty$, suy ra $a_n \to 1$, suy ra $x_n \to 1$.

Lưu ý: Bài này đã xuất hiện trong cuốn sách của thầy Chung. Lời giải trên cũng dựa vào lời giải trong sách. Thêm nữa, cho đặt $b_n$, không nhất thiết phải là con số $4$, theo ngu ý của mình thì tác giả đặt như thế để khi khử đi cho gọn thôi. Còn nếu bạn đặt số khác, muốn gọn như trên thì chắc thêm một bước đặt nữa là được. Chú ý công thức $\sin 2a = 2\sin a \cos a$ để tìm ra cách đặt trên.
[RIGHT][I][B]Nguồn: MathScope.ORG[/B][/I][/RIGHT]
 

thay đổi nội dung bởi: luxubuhl, 01-10-2013 lúc 06:21 PM Lý do: Tự động gộp bài
luxubuhl is offline   Trả Lời Với Trích Dẫn
The Following 4 Users Say Thank You to luxubuhl For This Useful Post:
Akira Vinh HD (24-11-2014), namdung (02-10-2013), Raul Chavez (03-10-2013), trungno (06-11-2013)
Old 01-10-2013, 06:46 PM   #7
n.t.tuan
+Thành Viên+
 
n.t.tuan's Avatar
 
Tham gia ngày: Nov 2007
Bài gởi: 1,250
Thanks: 119
Thanked 616 Times in 249 Posts
Đang thắc mắc, đã thấy ông anh lập topic rồi.
[RIGHT][I][B]Nguồn: MathScope.ORG[/B][/I][/RIGHT]
 
__________________
T.
n.t.tuan is offline   Trả Lời Với Trích Dẫn
The Following 2 Users Say Thank You to n.t.tuan For This Useful Post:
Akira Vinh HD (24-11-2014), namdung (02-10-2013)
Old 01-10-2013, 07:05 PM   #8
pco
+Thành Viên+
 
pco's Avatar
 
Tham gia ngày: Dec 2011
Bài gởi: 528
Thanks: 560
Thanked 195 Times in 124 Posts
Tổ hợp.
Bài 9. (Cần Thơ) Cho một bảng ô vuông không giới hạn số dòng và số cột. Ban đầu người ta viết hai số 1 và 3 vào hai ô khác nhau của bảng. Tiếp theo, người ta điền vào các ô trống các số nguyên dương theo quy tắc sau: Nếu trên bảng đã có 2 số a, b thì người ta có thể điều số c = a + b + ab vào một ô còn trống nào đó trên bảng. Hỏi bằng cách đó trên bảng có thể xuất hiện được các số 2509 và 20132014 hay không? Giải thích câu trả lời.

Lời giải. Bài toán này cũng tương tự bài toán $6$, chỉ cần chú ý một cái là $ab+a+b=(a+1)(b+1)-1$.
Với hai số $1$ và $3$ trên bảng thì số tiếp theo sẽ là $c=(1+1)(3+1)-1$. Số tiếp theo nữa có thể là $(1+1)(1+1)(3+1)-1$ (nếu ghép $c$ với $1$) hoặc $c$ hoặc $(1+1)(3+1)(3+1)-1$ (nếu ghép $c$ với $3$).
Cứ như thế thì các số được đưa vào bảng sẽ luôn có dạng $(1+1)^m(1+3)^n-1=2^k-1$.
Với hai số $2509$ và $20132014$ đều không có dạng trên nên trên bảng không thể xuất hiện hai số này. $\blacksquare$

[RIGHT][I][B]Nguồn: MathScope.ORG[/B][/I][/RIGHT]
 
__________________
"People's dreams... will never end!" - Marshall D. Teach.
pco is offline   Trả Lời Với Trích Dẫn
The Following 5 Users Say Thank You to pco For This Useful Post:
Akira Vinh HD (24-11-2014), giabao185 (06-10-2013), hoangqnvip (02-10-2013), Raul Chavez (03-10-2013), Saruka 01 (27-10-2013)
Old 01-10-2013, 09:06 PM   #9
quocbaoct10
+Thành Viên Danh Dự+
 
quocbaoct10's Avatar
 
Tham gia ngày: Oct 2012
Đến từ: THPT chuyên Lê Quý Đôn-Nha Trang-Khánh Hòa
Bài gởi: 539
Thanks: 292
Thanked 365 Times in 217 Posts
TỔ HỢP
Cách làm khác cho bài 9:
bài toán vẫn đúng nếu ta thay 2 số 1 và 3 bới 2 số lẻ bất kì.
ta đặt $a=2k+1$ và $b=2k'+1$, khi đó:
$c=a+b+ab=2(k+k')+2+(2k+1)(2k'+1)=4(k+k')+4kk'+3$, từ đó $c$ sẽ là số lẻ. Và tiếp tục như đến vô hạn lần thì các số xuất hiện trên bảng chỉ xuất hiện các số lẻ và chúng chia 4 dư 3 nên số 20132014 không xuất hiện trên bảng vì đây là 1 số chẵn và số 2509 cũng không xuất hiện trên bảng vì nó chia 4 dư 1 (đpcm).
3. (PTNK) Trong một hội nghị khoa học có 5000 đại biểu tham dự, mỗi một đại biểu biết duy nhất một thứ tiếng. Một ủy ban gồm một số đại biểu được gọi là ủy ban làm việc nếu tất cả các thành viên ủy ban đều biết chung một thứ tiếng, và được gọi là ủy ban thách thức không có hai thành viên nào của ủy ban biết chung một thứ tiếng (ủy ban có thể gồm 1 thành viên, ủy ban này gọi là làm việc cũng được, thách thức cũng được). Chứng minh rằng có thể chia các đại biểu thành đúng 100 ủy ban rời nhau (mỗi đại biểu thuộc đúng một ủy ban) sao cho các ủy ban này hoặc là ủy ban làm việc, hoặc là ủy ban thách thức.

Có lẽ là đưa về bài toán tổng quát: có thể chia $2n^2$ người thành $2n$ nhóm thảo ycđb. CHứng minh bằng quy nạp. các TH $n=1$, $n=2$ đúng (có thể tự kiểm chứng). Giả sử $n$ đúng, ta chứng minh đúng với $n+1$. đầu tiên, từ $2n^2+4n+2$ có thể chọn ra được $4n+2$ người sao cho:
Th 1: có $i$ người biết chung 1 thứ tiếng A, $4n+2-i$ còn lại biết chung thứ tiếng B (chọn trong TH tồn tại 1 ngôn ngữ mà có nhiều hơn $2n+1$) người cùng biết.
Th 2: có $i$ người biết chung 1 ngôn ngữ, $4n+2-i$ còn lại không biết chung 1 ngôn ngữ nào (trong TH không có ngôn ngữ nào có nhiều hơn $2n+1$ người biết chung ngôn ngữ).
Sau khi thực hiện chia ra thành hai nhóm như thế, ta sẽ chia $2n^2$ người này thành $n$ nhóm thỏa ycđb, còn với $4n+2$ người còn lại ta chia ra thành 2 nhóm, vậy từ $2n^2+4n+2$ người, ta đã chia thành $2(n+1)$ nhóm (đpcm). $n=50$ chính là bài toán của ptnk.
Lời giải này của em có lẽ còn nhiều chỗ chưa ổn định, đặc biệt là chỗ chứng minh quy nạp có thể chọn ra $4n+2$ từ $2n^2+4n+2$, mong mọi người giúp đỡ.
5. (Đà Nẵng) Cho bảng kẻ ô vuông kích thước (2n)×(2n+1). Hãy tìm giá trị lớn nhất của k sao cho k thoả mãn điều kiện: ta có thể tô màu k ô vuông đơn vị của bảng sao cho không có hai ô vuông đơn vị nào được tô mà có đỉnh chung.

chia $2n(n+1)$ thành $n(n+1)$ bảng $2 \times 2$, khi đấy, mỗi ô vuống này chỉ có thể chứa nhiều nhất 1 ô vuông tô màu vì nếu có 2 ô vuông được tô màu tồn tại trong 1 hình vuông 2x2 thì 2 ô vuông đó chung đỉnh (vô lí), vậy $k \le n^2+n$.
cách tô với $k=n^2+n$:
tô các ô $(2i+1, 2j+1), i=0,1,...,n-1; j=0,1,...,n$
------------------------------
8. (Lương Thế Vinh, Đồng Nai) Trong một thư viện người ta quan sát thấy được:
- Mỗi ngày có 5 người đọc sách.
- Hai ngày bất kì thì số người đọc sách là 9.
Hãy tính xem trong 1 tháng có bao nhiêu người đến đọc sách.Biết tháng đó có 30 ngày.
Phát biểu lại:
cho 30 tập $A_1,A_2,...,A_{30}$ thỏa
$|A_i|=5$ ($1\le i \le 30$)
$|A_i \cup A_j|=9$ ($1 \le i<j \le 30$)
tính số phần tử của $\cup_{i=1}^{30}A_i$.
lời giải
ta có $|A_i \cap A_j|=|A_i|+|A_j|-|A_i \cap A_j|=1$
vì $A_1$ có phần tử chung với 29 tập còn lại nên tồn tại phần tử a sao cho thuộc ít nhất 6 tập hợp khác ( nếu mỗi phần tử của A chỉ thuộc 5 tập thì số tập hợp bằng 5.5=25 < 30, vô lí).
vậy $a$ thuộc các tập $A_1,A_2,...,A_7$ .
ta chứng minh $a$ thuộc tất cả các tập còn lại.
VÌ 2 tập tùy ý không có phần tử chung nên các tập $A_1,A_2,...,A_7$ không có phần tử chung khác $a$.
Thật vậy, nếu tồn tại $A_k$ ($30 \ge k>7$) không chứa $a$ thì với mỗi $A_i$ ($1 \le i \le 7$), $B$ phải có phần tử chung $a_i$ khác $a$ và khác nhau với từng tập đó (nếu $a_i=a_j$ thì tồn tại 2 tập có 2 phần tử chung). Từ đấy, $A_k$ sẽ phải có ít nhất 7 phần tử (điều này vô lí). Do đó a thuộc $A_k$. VÌ $A_k$ là 1 tập bất kì trong $23$ tập còn lại nên $a$ thuộc 29 tập đã cho, từ đó $|\cup_{i=1}^{30}A_i|=(\sum_{i=1}^{30}(|A_i| -1)) +1=121$.
[RIGHT][I][B]Nguồn: MathScope.ORG[/B][/I][/RIGHT]
 
__________________
i'll try my best.

thay đổi nội dung bởi: quocbaoct10, 01-10-2013 lúc 09:45 PM Lý do: paste nhầm @@
quocbaoct10 is offline   Trả Lời Với Trích Dẫn
The Following 6 Users Say Thank You to quocbaoct10 For This Useful Post:
Akira Vinh HD (24-11-2014), hoangqnvip (02-10-2013), namdung (02-10-2013), pco (01-10-2013), Raul Chavez (03-10-2013), Saruka 01 (27-10-2013)
Old 01-10-2013, 09:39 PM   #10
Fool's theorem
+Thành Viên Danh Dự+
 
Fool's theorem's Avatar
 
Tham gia ngày: Oct 2012
Đến từ: T1 K46 Chuyên ĐHSP Hà Nội
Bài gởi: 187
Thanks: 42
Thanked 192 Times in 101 Posts
Gửi tin nhắn qua Yahoo chát tới Fool's theorem
Tổ hợp
Bài 10: Trên bàn cờ vua có một số quân cờ. Biết rằng nếu một ô nào đó còn trống thì tổng số lượng những quân cờ đứng cùng hàng và cùng cột với ô đó không nhỏ hơn 8. Chứng minh rằng trên bàn cờ đó có ít nhất 32 quân cờ.

Xét hàng có số quân cờ nhỏ nhất trong tất cả các hàng và các cột( nếu số quân cờ nhỏ nhất nằm vào một cột thì ta chỉ việc quay bàn cờ 90 độ)
Giả sử hàng đó có $k$ quân cờ. Hàng đó có $8-k$ ô trống nên $8-k$ cột chứa $8-k$ ô trên hàng này mỗi cột sẽ chứa ít nhất $8-k$ quân cờ.
Xét $k$ cột còn lại, mỗi cột chứa ít nhất $k$ quân cờ ( do k là số quân cờ ít nhất trên 1 hàng/cột).
Vậy ta có tổng số quân cờ sẽ lớn hơn hoặc bằng $(8-k)^2+k^2 \geq \frac{(8-k+k)^2}{2} =32$

Mở rộng :Bằng lý luận tương tự ta có thể xử lí bài toán tổng quát:
Trên bàn cờ $n*n$ có một số quân cờ. Biết rằng nếu một ô nào đó còn trống thì tổng số lượng những quân cờ đứng cùng hàng và cùng cột với ô đó không nhỏ hơn $n$. Chứng minh rằng trên bàn cờ đó có ít nhất $\frac{n^2}{2}$ quân cờ.
---------------------------------
Bài 7 : Ta đại số hóa bài toán :
Gọi các điểm được tô bởi màu xanh là $x_1,x_2,....x_k$
Dễ thấy một cách tô màu phụ thuộc hoàn toàn vào các điểm được tô màu xanh, nên số cách tô màu khác nhau chính là số bộ $x_1,x_2,....x_k$ thỏa mãn các điều kiện sau :
$1 \leq x_1 < x_2 < ....<x_k \leq n$ ; $x_{i+1} - x_i \geq (p+1)$, $x_k \leq n-p$
Và số bộ $x_1,x_2,....x_k$ thỏa mãn những điều kiện trên chính là số bộ $y_1,y_2,....y_k$ thỏa mãn:
$1\leq y_1 <y_2 < ...< y_k \leq n-kp$ với $y_i = x_i - (i-1)p$
Số bộ này chính là $C^k_{n-kp}$
[RIGHT][I][B]Nguồn: MathScope.ORG[/B][/I][/RIGHT]
 
__________________
Hope against hope.

thay đổi nội dung bởi: Fool's theorem, 01-10-2013 lúc 10:01 PM
Fool's theorem is offline   Trả Lời Với Trích Dẫn
The Following 6 Users Say Thank You to Fool's theorem For This Useful Post:
Akira Vinh HD (24-11-2014), hoangqnvip (02-10-2013), pco (01-10-2013), proboyhinhvip (01-10-2013), Raul Chavez (03-10-2013), vinh7aa (02-10-2013)
Old 02-10-2013, 11:24 AM   #11
nguyentatthu
Super Moderator
 
Tham gia ngày: Nov 2007
Đến từ: BH
Bài gởi: 212
Thanks: 135
Thanked 345 Times in 92 Posts
Tiếp theo là phần hình học.
[RIGHT][I][B]Nguồn: MathScope.ORG[/B][/I][/RIGHT]
 
File Kèm Theo
Kiểu File : doc HinhHoc VMO2014.doc (256.0 KB, 581 lần tải)
nguyentatthu is offline   Trả Lời Với Trích Dẫn
The Following 5 Users Say Thank You to nguyentatthu For This Useful Post:
Akira Vinh HD (24-11-2014), hoangqnvip (11-10-2013), mathandyou (02-10-2013), Raul Chavez (03-10-2013), trungno (06-11-2013)
Old 02-10-2013, 12:32 PM   #12
nguyentatthu
Super Moderator
 
Tham gia ngày: Nov 2007
Đến từ: BH
Bài gởi: 212
Thanks: 135
Thanked 345 Times in 92 Posts
Đây là phần số học
[RIGHT][I][B]Nguồn: MathScope.ORG[/B][/I][/RIGHT]
 
File Kèm Theo
Kiểu File : doc VMO 2014 Sohoc.doc (53.0 KB, 448 lần tải)
nguyentatthu is offline   Trả Lời Với Trích Dẫn
The Following 4 Users Say Thank You to nguyentatthu For This Useful Post:
Akira Vinh HD (24-11-2014), hoangqnvip (11-10-2013), namdung (02-10-2013), Raul Chavez (03-10-2013)
Old 02-10-2013, 02:23 PM   #13
hoangqnvip
+Thành Viên+
 
Tham gia ngày: Nov 2011
Đến từ: Quy Nhơn-Bình Định
Bài gởi: 66
Thanks: 283
Thanked 87 Times in 25 Posts
Gửi tin nhắn qua Yahoo chát tới hoangqnvip
Đây là một số bài mình đã giải trên VMF, mong các bạn xem thử

Bài 2:
Xét tích: $A=\left ( \frac{1}{1}+1 \right )...\left ( \frac{1}{2013}+1 \right )$
Sau khi thực hiện mỗi phép biến đổi, ta có tích trên không đổi
Vậy số cuối cùng sau 2012 lần biến đổi là $\left ( \frac{1}{1}+1 \right )...\left ( \frac{1}{2013}+1 \right )-1=2013$
Bài 4:
Bổ đề: số cách tô màu cho đa giác đều 12 cạnh sao cho không có mẫu đơn sắc nào là $906$
Xét đa giác đều 12 cạnh:
Ta chỉ cần đếm số cách tô màu không có tam giác đều và tứ giác đều nào cùng màu (có 4 tam giác đều và 3 tứ giác đều)
Có $6^4$ cách tô sao cho không có tam giác đều nào cùng màu
Xét các cách tô sao cho có ít nhất $1$ tứ giác đều cùng màu: Có $3\times 2\times 3^4$ cách tô
Xét các cách tô sao cho có ít nhất $2$ tứ giác đều cùng màu: Có $3\times \left(2+2\times 2^4 \right)$ cách tô
Xét các cách tô sao cho có $3$ tứ giác đều cùng màu: Có $6$ cách tô
Theo nguyên lí bù trừ ta có tổng cộng $906$ cách tô
Quay lại bài toán:
Ta nhận thấy rằng số cách tô màu 24-giác đều sao cho không có một mẫu đơn sắc nào cungx chính là số cách tô màu $2$ 12-giác đều không có một mẫu đơn sắc nào được tạo bởi $24$ đỉnh trên: $906^2$ cách tô (áp dụng bổ đề trên)
Bài 5:
Ta sẽ CM $k \leq n \left(n+1 \right)$
Xét $k \leq n \left(n+1 \right)$
Ta sẽ tô màu như sau:
Ở hàng 1 ta tô các ô thứ $1,3,5,..,2n+1$
Ở hàng 3 ta tô các ô thứ $1,3,5,..,2n+1$
...
Ở hàng 2n-1 ta tô các ô thứ $1,3,5,..,2n+1$
Ta sẽ CM $k<n^2+n+1$ bằng quy nạp
Xét $n=1$: dễ dàng suy ra đpcm
Giả sử BĐT trên đúng với $n=i$
Xét $n=i+1$:
Ta chia bảng trên thành 4 phần: $2i\times \left ( 2i+1 \right )$, $2\times 2i$, $2\times \left ( 2i+1 \right )$, $2\times 2$
Dễ dàng thấy rằng trong bảng $2i\times \left ( 2i+1 \right )$ số ô được tô màu bé hơn $i^2+i+1$, bảng $2\times 2i$ số ô được tô màu bé hơn hoặc bằng $i$,bảng $2\times \left ( 2i+1 \right )$ số ô được tô màu bé hơn hoặc bằng $i+1$, bảng $2\times2$ số ô được tô màu bé hơn hoặc bằng $1$
Suy ra $k<\left ( i+1 \right )^2+\left ( i+1 \right )+1$
Suy ra đpcm
Vậy $max_{k}=n\left ( n+1 \right )$
Bài 6:
Giả sử các số đó là $a_1<a_2<...<a_39$. Xét 20 số hạng đầu tiên sẽ tồn tại 2 số có chữ số tận cùng là 0 và sẽ có một trong 2 số có chữ số hàng chục khác 9. Gọi số đó là $n$
Xét 19 số $n+1,n+2,...,n+19$, ta có:
$S\left ( n+i \right )=S\left ( n \right )+i$ với $i=1,...,9$
Và $S\left ( n+19 \right )=S\left ( n \right )+10$
Ta có dãy số $S\left ( n \right )+1,...,S\left ( n \right )+10$
Theo nguyên tắc Dirichlet thì một trong các số sau phải chia hết cho 11, suy ra đpcm
Bài 7:
Gọi $x_i$ là khoảng cách giữa điểm tô màu đỏ thứ $i-1$ và $i$
Đặt:
$a_1=x_1$
$a_2=x_2-p$
$a_3=x_3-p$
$a_{k+1}=x_{k+1}-p$
Ta có số cách tô màu thoả mãn yêu cầu đề bài là số nghiệm nguyên không âm của PT:
$a_1+a_2+...+a_{k+1}=n-k\left ( p+1 \right )$
Áp dụng bài toán chia kẹo Euler, ta tính được bằng $C_{n-p\left ( k+1 \right )+k}^{k}$
Bài 8:
Gọi tập hợp người đọc của ngày thứ $i$ là $A_i$
Bổ đề: $\left | \bigcap_{i=1}^{30}A_i \right |=1$
Xét tập $A_1$:
Gọi $a$ là phần tử của $A_1$ thuộc nhiều tập nhất. Số tập hợp mà $a$ thuộc lớn hơn hoặc bằng 6
Gọi các tập hợp chứa $a$ là $A'_1,...,A'_m$ ($m \geq 6$)
Giả sử tồn tại tập $B$ không có phần tử $a$.
Mà $\left | B \right |=5$ nên theo nguyên tắc Dirichlet, có ít nhất 2 tập $A'_i,A'_j$ sao cho $\left | B\cap A'_i \cap A'_j \right |=1$
Mà $A'_i \cap A'_j =a$ nên $a \in B$(vô lí)
Suy ra $\bigcap_{i=1}^{30}A_i=\left \{ a \right \}$ (đpcm)
Quay lại bài toán:
Vì $\left | \bigcap_{i=1}^{30}A_i \right |=1$ nên dễ dàng thấy rằng $\left | \bigcup_{i=1}^{30}A_i \right |=30.4+1=121$
Bài 9:
Gọi $a_1,a_2,...$ là các số trên bảng
Xét tích sau:
$A=\left ( a_1+1 \right )\left ( a_2+1 \right )...\left ( a_n+1 \right )$
Nhận xét: Mọi số $a_i+1$ đều có dạng $2^k$
Mặt khác, $2509+1$ và $20132014+1$ không có dạng này nên không thể xuất hiện, suy ra đpcm
Bài 10:
Gọi $k$ là số các ô còn trống.
Giả sử $k \geq 33$
Ta cần CM trong một hàng (hoặc cột) không tồn tại 6 ô vuông trống
Giả sử tồn tại 6 ô vuông trống trên một hàng.
Khi đó trên mỗi cột có ít nhất 6 ô có quân cờ, suy ra $k \leq 28$, vô lí
Vậy trong một hàng (hoặc cột) có nhiều nhất 5 ô vuông trống
Vì $k \geq 33$ nên theo nguyên tắc Dirichlet tồn tại một hàng có 5 ô vuông trống.
Tương tự, tồn tại một cột có 5 ô vuông trống.
Hàng đó và cột đó giao nhau tại một ô, ô đó phải là ô có cờ
Gọi các hàng chứa 5 ô ở cột đó là $r_1,r_2,...,r_5$
Gọi các cột chứa 5 ô ở hàng đó là $c_1,c_2,...,c_5$
Dễ thấy có ít nhất $30$ ô chứa quân cờ
Gọi ô được giao bởi cột $i$ và hàng $j$ là $a_{ij}$ và ta biểu diễn $a_{ij}=1$ nếu ô $a_{ij}$ chứa quân cờ, ngược lại $a_{ij}=0$
Nhận xét: có nhiều nhất một $a_{ij}=0$ (nếu có 2 $a_{ij}=0$ thì $k \leq 32$, mâu thuẫn)
Vậy có ít nhất một cột trong $8$ cột không chứa bất kì quân cờ nào trong $30$ quân cờ trên.
Vì số ô chưa đặt quân cờ này không vượt quá $5$ nên ta có ít nhất 3 ô có quân cờ.
Suy ra có ít nhất $33$ ô vuông có quân cờ, mâu thuẫn
Suy ra đpcm
[RIGHT][I][B]Nguồn: MathScope.ORG[/B][/I][/RIGHT]
 
File Kèm Theo
Kiểu File : doc To hop sol.doc (47.5 KB, 342 lần tải)
hoangqnvip is offline   Trả Lời Với Trích Dẫn
The Following 5 Users Say Thank You to hoangqnvip For This Useful Post:
Akira Vinh HD (24-11-2014), cool hunter (06-10-2013), luugiangnam (02-10-2013), pco (02-10-2013), Raul Chavez (03-10-2013)
Old 02-10-2013, 02:56 PM   #14
pco
+Thành Viên+
 
pco's Avatar
 
Tham gia ngày: Dec 2011
Bài gởi: 528
Thanks: 560
Thanked 195 Times in 124 Posts


Bài 1. Chứng minh rằng với mọi số nguyên tố $p$ thì $p^3+ \frac{p-1}{2}$ không là tích hai số tự nhiên liên tiếp.
Lời giải.
Bình luận.
[RIGHT][I][B]Nguồn: MathScope.ORG[/B][/I][/RIGHT]
 
__________________
"People's dreams... will never end!" - Marshall D. Teach.

thay đổi nội dung bởi: pco, 02-10-2013 lúc 03:00 PM
pco is offline   Trả Lời Với Trích Dẫn
The Following 4 Users Say Thank You to pco For This Useful Post:
Akira Vinh HD (24-11-2014), hoangqnvip (02-10-2013), Raul Chavez (03-10-2013), thaygiaocht (02-10-2013)
Old 02-10-2013, 06:20 PM   #15
pco
+Thành Viên+
 
pco's Avatar
 
Tham gia ngày: Dec 2011
Bài gởi: 528
Thanks: 560
Thanked 195 Times in 124 Posts
Bài 9. Tìm tất cả các số nguyên tố $p$ sao cho tồn tại số nguyên dương $n$ mà số $A=2+n+n^2+n^3+ \cdots + n^{p-1}$ là lũy thừa bậc $5$ của một số nguyên dương.

Bổ đề. Nếu $p,q$ là hai số nguyên tố và $n \ge 2$ là số nguyên dương thỏa mãn $q|\dfrac{n^p-1}{n-1}$ thì hoặc $p=q$ hoặc $q \equiv 1 \pmod{p}$.

Chứng minh.

Lời giải.
Bình luận.
[RIGHT][I][B]Nguồn: MathScope.ORG[/B][/I][/RIGHT]
 
__________________
"People's dreams... will never end!" - Marshall D. Teach.
pco is offline   Trả Lời Với Trích Dẫn
The Following 4 Users Say Thank You to pco For This Useful Post:
Akira Vinh HD (24-11-2014), cool hunter (06-10-2013), hoangqnvip (02-10-2013), Raul Chavez (03-10-2013)
Trả lời Gởi Ðề Tài Mới

Bookmarks


Quuyền Hạn Của Bạn
You may not post new threads
You may not post replies
You may not post attachments
You may not edit your posts

BB code is Mở
Smilies đang Mở
[IMG] đang Mở
HTML đang Tắt

Chuyển đến


Múi giờ GMT. Hiện tại là 01:10 AM.


Powered by: vBulletin Copyright ©2000-2024, Jelsoft Enterprises Ltd.
Inactive Reminders By mathscope.org
[page compression: 144.26 k/161.71 k (10.79%)]